Массовая перенормировка: геометрическая серия одночастичных неприводимых диаграмм

Почти везде, куда бы я ни посмотрел, утверждается, что полная двухточечная функция Грина (скажем, для поля Клейна-Гордона) представляет собой геометрическую серию в одночастичных неприводимых диаграммах, т.е. в импульсном пространстве,

г ( к ) "=" г 0 ( к ) + г 0 ( к ) Σ ( к ) г 0 ( к ) + г 0 ( к ) Σ ( к ) г 0 ( к ) Σ ( к ) г 0 ( к ) +
"=" г 0 ( к ) ( 1 + Σ ( к ) г 0 ( к ) + Σ ( к ) г 0 ( к ) Σ ( к ) г 0 ( к ) + )

и что сумма этого, используя формулу геометрического ряда,

г ( к ) "=" г 0 ( к ) 1 Σ ( к ) г 0 ( к ) "=" 1 к 2 + м 2 Σ ( к ) я ϵ

(например, на первой странице в правом верхнем углу здесь или внизу/вверху страниц 56/57 здесь ).

Однако какое основание для суммирования этого геометрического ряда таким образом? Это никогда не кажется оправданным, и не кажется, что | Σ ( к ) г 0 ( к ) | < 1 . Даже если я перенормирую массу м так что м р 2 "=" м 2 Σ ( к ) конечно, если суммирование геометрических рядов не обосновано, то бесконечности не сократятся и все равно все будет расходиться. Происходит ли неявное суммирование? Является ли этот шаг полностью непертурбативным?

Е д я т 1 : В основном, как я это вижу, ситуация такова: я спрашиваю, какова амплитуда пропагатора (в импульсном пространстве), и вы говорите

г 0 ( к ) 1 Σ ( к ) г 0 ( к ) "=" 1 к 2 + м р 2 я ϵ
где м р 2 "=" м 2 Σ ( к ) (конечная) масса. Затем я замечаю, что вы можете приблизительно вычислить его, используя ряд возмущений, заданный теорией, т.е.
г 0 ( к ) ( 1 + Σ ( к ) г 0 ( к ) + Σ ( к ) г 0 ( к ) Σ ( к ) г 0 ( к ) + ) .
Однако я замечаю, что Σ ( к ) г 0 ( к ) > 1 и я знаю, что для Икс > 1 ряд возмущений, заданный
1 + Икс + Икс 2 +
не является хорошим приближением к 1 1 Икс , при любом порядке ряда возмущений. Таким образом, может показаться, что теория возмущений терпит неудачу, потому что она не дает хорошего приближения ни в каком порядке для пропагатора. Итак, тогда я спрашиваю, каково оправдание всего этого в первую очередь? Они просто видят, что ряд возмущений для пропагатора такой же, как и ряд возмущений для 1 1 Икс , но понимая, что | Икс | > 1 а затем предполагая, что фактический пропагатор должен быть 1 1 Икс ? Потому что это, казалось бы, совершенно непертурбативный шаг.

Е д я т 2 : Я собираюсь сделать фактические вычисления, которые меня смущают, и если кто-то может указать, где это неправильно (если это так), это будет большим подспорьем. Кстати, ранее я делал вращение фитиля, но в этот раз не буду этого делать:

У нас есть это

г 0 ( к ) 1 Σ ( к ) г 0 ( К ) "=" 1 к 2 м р 2

где м р - физическая (и конечная) масса, и где г 0 ( к ) "=" 1 к 2 м 2 где м - голая масса, зависящая от отсечки. Переставляя, я получаю

Σ ( к ) г 0 ( к ) "=" 1 г 0 ( к ) ( к 2 м р 2 ) "=" 1 к 2 м р 2 к 2 м 2 .
Давайте посмотрим на регион, где м р 2 < к 2 < м 2 . м 2 так как отсечка удалена, так что это очень большая область. Теперь мне кажется, что в приведенном выше выражении для Σ ( к ) г 0 ( к ) , что правая часть ВСЕГДА больше единицы и даже, возможно, близка к бесконечности для определенных значений к (или, может быть, он не получает значений, близких к бесконечности, потому что к должно быть ограничено значениями ниже отсечки, но это не очень важно). Это, казалось бы, делает все геометрическое суммирование подозрительным. Есть ли ошибка?

Если у меня есть базовое непонимание того, как это работает, я хотел бы знать, что это самый простой случай перенормировки в QFT, но я этого не понимаю.

Эти вещи вряд ли когда-либо оправдываются в литературе. Думаю, лучшее апостериорное обоснование состоит в том, что это просто работает: пертурбативная КТП дает разумные результаты на н -loop level, но я надеюсь, что кто-нибудь ответит на этот вопрос, чтобы дать лучший.
В таких случаях физики-теоретики любят говорить об асимптотическом расширении. Но я также надеюсь, что кто-то в этой области ответит более подробно и разъяснит проблему.
Я не уверен, что вы знаете, что два распространителя 1 к 2 + м 2 и 1 к 2 м 2 не являются частью, вращаемой фитилем, но на самом деле являются пропагаторами в двух разных метрических соглашениях, используемых в литературе. Они написаны так, что к 0 имеет полюса | к | 2 + м 2 на реальной оси. Проблема, которую вы описываете, является просто результатом неудачного анализа, который вы сделали, и также может появиться с 1 к 2 + м 2 . Я обращусь к этому в дальнейших комментариях.
Использование двух разных метрических соглашений не изменит никакой физики, и поэтому Σ ( к ) будет такое же поведение. В первом случае у вас было м р 2 "=" м 2 Σ ( к ) так как физическая масса в результате голая масса должна уйти в + отменить + приходящий из Σ ( к ) чтобы получить конечный результат. Во втором случае голая масса должна уйти в отменить + приходящий из Σ ( к ) в ( м 2 + Σ ( к ) ) поскольку изменение метрического соглашения не изменится Σ ( к ) поведение.
Это в основном привело бы к м р 2 < к 2 < , что не имеет смысла, и вы можете изменить его на < к 2 < м р 2 . Если к 2 всегда больше, чем физическая масса, тогда она никогда не появится как полюс в пропагаторе, и вы никогда не сможете генерировать физическую массу с помощью радиационной поправки, в результате все ваши расчеты напрасны, и поэтому я рассматривал k в пределе высоких энергий в ответ.
Предупреждение: если вы пытаетесь доказать что-то неправильное, что делается в каждой книге, то вы должны иметь в виду, что в какой-то момент вы должны делать некоторые ошибки, и даже если некоторые люди здесь и там говорят вам, что это необоснованно, это должно быть быть оправданным. Та же проблема есть и в другом пропагаторе, если учесть к 2 < м р 2 . Удачи.
Небольшой комментарий к сообщению (v12): Пожалуйста, рассмотрите возможность явного указания автора, названия и т. д. ссылок, чтобы можно было восстановить ссылку в случае ее порчи.

Ответы (4)

Вы задаете законный вопрос. Мы можем перевернуть всю ситуацию с ног на голову. Вместо того, чтобы начинать с геометрического ряда

г ( к ) "=" г 0 ( к ) + г 0 ( к ) Σ ( к ) г 0 ( к ) + г 0 ( к ) Σ ( к ) г 0 ( к ) Σ ( к ) г 0 ( к ) + , ( 1 )
и прибытие в
г ( к ) "=" г 0 ( к ) 1 Σ ( к ) г 0 ( к ) "=" 1 к 2 + м 2 Σ ( к ) я ϵ , ( 2 )
мы можем пойти наоборот. Вместо этого мы будем утверждать, что ур. (2) справедливо и вызывает сомнительное расширение уравнения. (1) под вопросом, и это правильно (ОП назвал глупость в предположении | Σ ( к ) г 0 ( к ) | < 1 ).

Если вы сделаете домашнее задание, покопавшись в литературе по QFT, вы обнаружите множество непертурбативных способов вывода уравнения. (2) без обращения к ур. (1). Пертурбативная КТП старой школы интуитивно понятна, но является математической катастрофой.

Можете ли вы дать ссылку на книгу, которая выводит это непертурбативным способом?
@Suriya, попробуйте любую книгу с подробным описанием уравнения Швингера-Дайсона и его приближенного решения первого порядка.

Пертурбативные расширения обычно расходятся в теории поля, поэтому я рискну предположить, что полное резюме неприводимых диаграмм с одной частицей на самом деле расходится.

Но это не проблема, если вы просто хотите перенормировать. Имейте в виду, что на практике вы всегда перенормируете в конечном порядке в своем параметре расширения, поэтому действительно не нужно беспокоиться о бесконечном пересуммировании. Сумма усеченного геометрического ряда хорошо определена, поэтому для перенормировки вы определяете контрчлены так, чтобы ваш пропагатор был конечным в том порядке, в котором вы выполняете свои вычисления.

Вопрос ОП заключался в следующем: как можно обосновать сумму полного геометрического ряда (которая расходится, поэтому выражение для суммы неверно) в реальных вычислениях. Я не сомневаюсь, что то, что вы написали, правда, но это не отвечает на вопрос ОП.
@SolenodonParadoxus Я ответил на это в том смысле, что сказал, что вся серия (скорее всего) расходится настолько строго, что это не оправдано. Когда вы видели пересуммирование геометрического ряда всех диаграмм 1PI в реальных вычислениях?
@AnarchistBirdsWorshipFungus Вы предполагаете, что если я установлю м 2 "=" м р 2 + Σ ( к ) для некоторых фиксированных м р , тогда ряд возмущений в некотором порядке является хорошим приближением к 1 к 2 + м р 2 я ϵ ? Потому что так не кажется, а если нет, то как обосновывается такое суммирование?
JLA знаете ли вы, что даже после перенормировки, когда вы получаете конечные вклады в каждом порядке по теории возмущений, и даже когда вы рассматриваете связь, намного меньшую единицы, радиус сходимости пертурбативных разложений равен нулю? это ключ к моей точке зрения. Полная сумма пертурбативного расширения (считается) расходится. Проверьте это физика.stackexchange.com/q/70411 . Поэтому, скорее всего, никогда нельзя суммировать все диаграммы, так что не беспокойтесь об этом.
Полный Σ после суммирования всех диаграмм даже добавление всех контрчленов (скорее всего) расходится.

Ваше основное непонимание заключается в том, что | Σ ( к ) г 0 ( к ) | > 1 , что неверно, и в основном используются три принципа, по крайней мере, чтобы убедиться, что | Σ ( к ) г 0 ( к ) | < 1 . Во-первых, это очень просто, это перенормируемость теории, а во-вторых, это локальный характер контрчленов. Третий ингредиент не нужен, за исключением случая скалярных частиц, что в стандартной модели называется «естественностью».

Срок коррекции массы Σ ( к ) представляет собой сумму бесконечного множества графиков, дающих вклад в собственную энергию частицы, которая в общем случае будет иметь линейную расходимость (как собственная энергия электрона) или квадратичную расходимость (как собственная энергия фотона) в пределе высоких энергий. В случае частиц, отличных от нулевого спина, можно использовать различные схемы регуляризации, чтобы сделать это поведение только логарифмически расходящимся, и переопределение параметра массы (перенормировка) поглощает это расхождение.

Это в основном говорит вам, что Σ ( к ) ведет себя как л н ( к / м ) с k можно заменить шкалой Λ и у вас будет | Σ ( к ) г 0 ( к ) | поведение как л н ( к ) / к 2 как

г 0 ( к ) "=" 1 к 2 + м 2

Сейчас л н ( к ) / к 2 всегда меньше, чем 1 (как л н ( к ) к 2 всегда отрицательно).

Однако в случае скалярных частиц у вас есть Σ ( к ) ведут себя столь же плохо, как и квадратично расходящиеся (недостаточно симметрий в теории), для чего нужно иметь много сокращений, включающих Λ 2 член, чтобы сгладить полученные интегралы, чтобы они не расходились так сильно, как квадратичные. Однако нет никаких оснований ожидать, что такие отмены должны происходить «естественным образом», поскольку вам необходимо отрегулировать теорию, чтобы получить надлежащую отмену, в которой вы можете иметь м п час у с я с а л 2 << Λ . В результате этих отмен вы смягчите поведение Σ ( к ) который не будет расходиться так сильно, как Λ 2 и у вас будет | Σ ( к ) г 0 ( к ) | < 1 как требуется.

Однако проблема тонкой настройки не является физическим решением и, следовательно, должна быть дополнена дополнительной симметрией теории. Примером этого является суперсимметрия, которая обеспечивает поведение симметрии суперпартнера со спином 1/2, чтобы сделать расхождение логарифмическим, но это другой вопрос.

Я надеюсь, что это объяснит все ваши опасения и попытается найти хорошую книгу по qft, такую ​​как qft Средницкого.

Помимо проблем, связанных с предметной областью, обратите внимание, что правая часть вашего первого уравнения является четко определенным оператором, если геометрический ряд с Вопрос "=" Б А 1 сходится. (взять, к примеру, А "=" 1 , Б "=" 2 ).
Ладно, смотри выше.
@ved Я ценю ваш ответ, вы можете увидеть мою правку на мой вопрос? Надеюсь, теперь стало более понятно.
@JLA Я удалил ненужные части и сделал их максимально понятными.
@ved Спасибо за ваш ответ. Вы видите "Редактировать 2" в моем посте? Я написал это во многом в ответ на ваш пост.

Вы можете рассматривать это как выборочное суммирование диаграмм и определение в конечном счете расходящегося ряда как конечной функции. Это было известно еще во времена раннего Эйлера, как суммировать такие ряды, когда формальный параметр разложения на самом деле не мал. Прочтите книгу Харди на эту тему.

Настоящая проблема не здесь. В классической электродинамике можно точно рассчитать обратную реакцию, и, увы, это расходящееся дополнение к массе частицы. Это связано с тем, что обратная реакция (самовоздействие) представляет собой в основном самоиндукцию точечного заряда. Никакая внешняя сила не может ускорить заряд из-за этой бесконечной самоиндукции. Обратите внимание: идея самодействия — это анзац, а не что-то бесспорное. Вот почему практически в любой теории приходится отбрасывать массовые поправки: даже конечные, они не нужны, потому что в исходных уравнениях у нас была наблюдаемая масса. Таким образом, отбрасывание (массовая перенормировка).

Кроме того, пропагатор электрона не является калибровочно-инвариантным.

Хуже того, в КЭД всегда практически равна единице вероятность испустить мягкие фотоны, потому что электрон постоянно связан с фотонными осцилляторами. В вашем упражнении это не учитывается; в КЭД это учтено позже. Электрон на самом деле является инфрачастицей (погуглите).